Question

If you have a 12 year old patient with iron levels of 100(ug/dL), you will need to prescribe an iron supplement. How muc...

If you have a 12 year old patient with iron levels of 100(ug/dL), you will need to prescribe an iron supplement. How much iron (in mg) needs to be administered to your patient to bring iron levels into the normal range?(150 ug/dL)

Please show all work.

0 0
Add a comment Improve this question Transcribed image text
Answer #1

Solution we have to maintain 150 mg but we have but we have to ug low ng - IL let idL is the solution, so amount of Iron pill

Add a comment
Know the answer?
Add Answer to:
If you have a 12 year old patient with iron levels of 100(ug/dL), you will need to prescribe an iron supplement. How muc...
Your Answer:

Post as a guest

Your Name:

What's your source?

Earn Coins

Coins can be redeemed for fabulous gifts.

Not the answer you're looking for? Ask your own homework help question. Our experts will answer your question WITHIN MINUTES for Free.
Similar Homework Help Questions
  • If you have a 12 year old patient with iron levels of 110 (ug/dL), you will...

    If you have a 12 year old patient with iron levels of 110 (ug/dL), you will need to prescribe an iron supplement. How much iron (in mg) needs to be administered to your patient to bring irons levels into the normal range? Normal range is 150 ug/dL. Blood Volume of the patient is 3.1 L

  • you have an adult male patient with iron levels of 220 ug/dL you may have to...

    you have an adult male patient with iron levels of 220 ug/dL you may have to use Deferoxamine to bind the iron and allow for excretion. Deferoxamine has a molar mass of 656.8 g/mol and binds to iron in a 1:1 mole ratio. How much deferoxamine in grams must be administered to your patient to bring iron levels into the normal range. patient has a blood volume of 4.7 L. normal range iron levels is 170 ug/dL

  • (12) A 45 year old male patient presents with the following lab values. What do you...

    (12) A 45 year old male patient presents with the following lab values. What do you conclude about his iron status? (2 points) HgB: 14.1 g/dL ok Serum Ferritin: 240 ug/LO RBC folate: 463 nmol/Lm/ Serum B12: 263 pmol/L Serum C-reactive protein: 26 mg/L a) His iron status is adequate, since ferritin concentrations are above the cutoff for deficiency b) You cannot interpret his plasma ferritin concentration because he has inflammation le may have iron overload, since ferritin concentrations are...

  • CASE STUDY 5 Mrs. C., a 79-year-old woman, presented to the emergency department barely able to...

    CASE STUDY 5 Mrs. C., a 79-year-old woman, presented to the emergency department barely able to walk. She said that she had gotten progressively weaker in the past couple of weeks and that she had noticed that her appetite was failing. She had seen some yellow color to her eyes and skin, and that worried her. She had no desire to eat but she did crave ice. Mrs. C. was thin, emaciated, and pale; she had difficulty walking and seemed...

  • 6. You have a 16 year old pregnant patient who is anemic and needs to be...

    6. You have a 16 year old pregnant patient who is anemic and needs to be started on iron supplementation. She asks you why she needs to take the iron. What do you need to tell her about iron and how the body uses iron to make RBCs?What other teaching is necessary regarding oral iron supplementation? (Consider drug-drug, drug-food interactions, tolerance, side effects, adverse effects, diet restrictions, etc.)

  • Case: You are a physician assistant taking care of a 65-year old retired man with a...

    Case: You are a physician assistant taking care of a 65-year old retired man with a history of cardiovascular disease, including two heart attacks. A thorough history and physical exam reveal: height= 170 cm, weight= 80 kg, BP= 150/90, HR= 100 bpm, LDL= 200 mg/dl, TG= 250 mg/dl, HDL= 30 mg/dl, FPG=80 mg/dl and HbA1c= 3%. As a grandfather, your patient spends most of his days taking care of his grandchildren while the parents, who both work full-time, are at...

  • Case: You are a physician assistant taking care of a 65-year old retired man with a...

    Case: You are a physician assistant taking care of a 65-year old retired man with a history of cardiovascular disease, including two heart attacks. A thorough history and physical exam reveal: height= 170 cm, weight= 80 kg, BP= 150/90, HR= 100 bpm, LDL= 200 mg/dl, TG= 250 mg/dl, HDL= 30 mg/dl, FPG=80 mg/dl and HbA1c= 3%. As a grandfather, your patient spends most of his days taking care of his grandchildren while the parents, who both work full-time, are at...

  • Case B: You are a physician assistant taking care of a 65-year old retired man with...

    Case B: You are a physician assistant taking care of a 65-year old retired man with a history of cardiovascular disease, including two heart attacks. A thorough history and physical exam reveal: height= 170 cm, weight= 80 kg, BP= 150/90, HR= 100 bpm, LDL= 200 mg/dl, TG= 250 mg/dl, HDL= 30 mg/dl, FPG=80 mg/dl and HbA1c= 3%. As a grandfather, your patient spends most of his days taking care of his grandchildren while the parents, who both work full-time, are...

  • ST, a 32-year-old patient was diagnosed with type 2 diabetes mellitus after the birth of her...

    ST, a 32-year-old patient was diagnosed with type 2 diabetes mellitus after the birth of her first child; her blood sugar was 180 mg/dL. Her serum glucose level has been maintained within the normal range with metformin 500 mg/day. Use the drug cards you created and your textbook to respond to the following questions: Why is ST taking an oral antidiabetic medication rather than insulin? When should metformin not be taken? Two years later, ST became pregnant again. Metformin was...

  • There is a 22 year-old female patient in E.R. You have been called down to do...

    There is a 22 year-old female patient in E.R. You have been called down to do a cardiopulmonary assessment on her. She currently on a 50% venti-mask, SPO2= 84, RR-3, HR-155, breath sounds clear but decreased; no current history, patient is awake, but work of breathing seems to be high. Please list issues that might suggest that the patient may need mechanical ventilation. Abg has been done, = 7.31, CO2-59, PaO2-49, HCO3-23. Does this patient need mechanical ventilation: Yes or...

ADVERTISEMENT
Free Homework Help App
Download From Google Play
Scan Your Homework
to Get Instant Free Answers
Need Online Homework Help?
Ask a Question
Get Answers For Free
Most questions answered within 3 hours.
ADVERTISEMENT
ADVERTISEMENT
ADVERTISEMENT